User avatar
 
ManhattanPrepLSAT1
Thanks Received: 1909
Atticus Finch
Atticus Finch
 
Posts: 2851
Joined: October 07th, 2009
 
This post thanked 6 times.
 
 

Re: Q19 - The coach of the Eagles

by ManhattanPrepLSAT1 Fri Dec 31, 1999 8:00 pm

The analysis reveals that the team has lost only when Jennifer was not playing. The conclusion reached is that if Jennifer plays, then the eagles will win.

The argument concludes something about the future about information collected from the past. It's not that team loses only when Jennifer does not play, but rather that the team has lost only when Jennifer was not playing. Thus the flaw, is that the argument assumes what is true of one time period will be true of another.

Here's an analogy...

If I hire someone who was very successful in their last position as a fundraiser, does that mean that the person will be a successful fundraiser for me.

or if

The Suns won their last basketball game, does that mean that they will win their next game.

This temporal flaw is identified in answer choice (D).

(A) is really close but should not have said that the factor was "sufficient." The words "only when" in the stimulus imply that the factor is necessary, not that it is sufficient.
(B) is not true. The argument never tries to nor assumes that a contribution can be quantified.
(C) is not true. The argument's conclusion is not about sports in general but rather the results of future Eagles' games.
(D) properly identifies the flaw.
(E) is not true. The conclusion is not about computer analyses.


#officialexplanation
 
nicole_mazga
Thanks Received: 0
Forum Guests
 
Posts: 3
Joined: May 26th, 2010
 
 
 

Q19 - The coach of the Eagles

by nicole_mazga Thu May 27, 2010 5:07 pm

I'm having some trouble with this one, I choose A. What makes D a better response?
 
linzru86
Thanks Received: 3
Forum Guests
 
Posts: 24
Joined: June 08th, 2010
 
 
 

Re: PT 54, S2, Q19 - The coach of the Eagles...

by linzru86 Thu Sep 16, 2010 2:58 pm

Ok but say that A did say necessary instead of sufficient, that would make sense correct? Im just trying to get the necessary/sufficient concept down pat.
User avatar
 
ManhattanPrepLSAT1
Thanks Received: 1909
Atticus Finch
Atticus Finch
 
Posts: 2851
Joined: October 07th, 2009
 
This post thanked 1 time.
 
 

Re: PT 54, S2, Q19 - The coach of the Eagles...

by ManhattanPrepLSAT1 Fri Sep 17, 2010 7:49 pm

The answer would be correct if it said

(A) infers from the fact that a certain factor is necessary for a result that the absence of that factor is sufficient for the opposite result

really close, huh?
 
mrudula_2005
Thanks Received: 21
Forum Guests
 
Posts: 136
Joined: July 29th, 2010
 
 
trophy
First Responder
 

Re: PT 54, S2, Q19 - The coach of the Eagles...

by mrudula_2005 Sat Oct 02, 2010 9:24 pm

mshermn Wrote:The answer would be correct if it said

(A) infers from the fact that a certain factor is necessary for a result that the absence of that factor is sufficient for the opposite result

really close, huh?


I understand this. But if you match (A) up, it lines up:

the author says: If lost --> J not play
If J plays --> will win

matching this up to (A), we can see it point by point: the author infers from the fact that a certain factor is sufficient for a result (losing sufficient for J not playing), that the absence of that factor (winning) is necessary for the opposite result (J playing).

What am I doing wrong here?


Also, isn't another flaw (in addition to (D)), that the coach says If lost --> J not play and then says J plays --> win, when in fact it technically should be If lost --> J not play and then J play --> not lose (i.e., they could tie).


THANK you in advance!
User avatar
 
LSAT-Chang
Thanks Received: 38
Atticus Finch
Atticus Finch
 
Posts: 479
Joined: June 03rd, 2011
 
 
trophy
Most Thankful
trophy
First Responder
 

Re: Q19 - The coach of the Eagles...

by LSAT-Chang Sun Sep 18, 2011 5:50 pm

Hey Matt, I would love to hear your answer to the above post. I correctly chose (D), but was definitely tempted by (A) but wasn't sure if I had it matched up correctly, so didn't pick it.

So we have:

Lose --> Jennifer NOT playing
Jennifer playing --> NOT lose

Actual answer choice (A): infers from the fact that a certain factor is sufficient for a result that the absence of that factor is necessary for the opposite result:

Lose is sufficient for Jennifer NOT playing
NOT lose is necessary for Jennifer playing

Your answer choice (A) that would have made it correct: infers from the fact that a certain factor is necessary for a result that the absence of that factor is sufficient for the opposite result

Jennifer NOT playing is necessary for Lose
Jennifer playing is sufficient for not Lose

Do you see my point?

The second question I had was, isn't there also a correlation-causation issue going on here? Just because in the past the team has lost only when Jennifer wasn't palying, you can't conclude that Jennifer's presence ENSURES (causation word) that they will win. Something else could have caused the team to lose and it just happened to be that on those days, Jennifer wasn't playing, right? I went to look for an answer choice that had a correlation-causation issue, but I wasn't able to find any, and when I read (D), the word "coincided" popped out at me, and so I knew that had to be the flaw.

I would love to hear your thoughts on this!
 
zainrizvi
Thanks Received: 16
Atticus Finch
Atticus Finch
 
Posts: 171
Joined: July 19th, 2011
 
This post thanked 1 time.
 
trophy
First Responder
 

Re: PT 54, S2, Q19 - The coach of the Eagles...

by zainrizvi Mon Oct 31, 2011 12:35 pm

mshermn Wrote:The answer would be correct if it said

(A) infers from the fact that a certain factor is necessary for a result that the absence of that factor is sufficient for the opposite result

really close, huh?



I don't think that would be right though. Right now it says:
L - > Jnp
The contrapositive being,
Jp -> W

Isn't that valid reasoning?
 
jennifer
Thanks Received: 0
Elle Woods
Elle Woods
 
Posts: 91
Joined: July 29th, 2010
 
 
 

Re: Q19 - The coach of the Eagles...

by jennifer Thu Nov 03, 2011 9:36 pm

In reference to the wording of answer choice a, can someone please translate this Into common words which i can understand and give an example or sinario of what this means.
User avatar
 
ManhattanPrepLSAT1
Thanks Received: 1909
Atticus Finch
Atticus Finch
 
Posts: 2851
Joined: October 07th, 2009
 
This post thanked 1 time.
 
 

Re: Q19 - The coach of the Eagles...

by ManhattanPrepLSAT1 Fri Nov 04, 2011 3:28 am

zainrizvi Wrote:mshermn wrote:
The answer would be correct if it said

(A) infers from the fact that a certain factor is necessary for a result that the absence of that factor is sufficient for the opposite result

really close, huh?



I don't think that would be right though. Right now it says:
L - > Jnp
The contrapositive being,
Jp -> W

Isn't that valid reasoning?


Hey zainrizvi! Nice work, you are absolutely correct! My explanation was incorrect. I should have caught that. Answer choice (A) would not have been correct had it been worded as I suggested since it still would have been valid reasoning. I think I was trying to describe what was happening in the argument and lost sight of the fact that we were looking for the error in the reasoning. The use of the contrapositive, which is essentially one element of the reasoning structure and that is highlighted in answer choice (A) is not a reasoning error and so cannot answer the question about the argument's vulnerability.

changsoyeon Wrote:The second question I had was, isn't there also a correlation-causation issue going on here? Just because in the past the team has lost only when Jennifer wasn't palying, you can't conclude that Jennifer's presence ENSURES (causation word) that they will win. Something else could have caused the team to lose and it just happened to be that on those days, Jennifer wasn't playing, right? I went to look for an answer choice that had a correlation-causation issue, but I wasn't able to find any, and when I read (D), the word "coincided" popped out at me, and so I knew that had to be the flaw.

So I don't see a correlation/causation issue going on here. "Ensures" is still a word that implies correlation.

mrudula_2005 Wrote:Also, isn't another flaw (in addition to (D)), that the coach says If lost --> J not play and then says J plays --> win, when in fact it technically should be If lost --> J not play and then J play --> not lose (i.e., they could tie).

I like this thinking, absolutely! It's possible that the team could simply tie!

jennifer Wrote:In reference to the wording of answer choice a, can someone please translate this Into common words which i can understand and give an example or sinario of what this means.

Answer choice (A) simply says that the argument makes a mistake by taking the contrapositive of a conditional relationship. But a contrapositive is not an error of reasoning.

Answer choice (A) says that the argument is flawed for saying

Team Loses ---> ~Jennifer Plays
-------------------------------------
Jennifer Plays ---> ~Team Loses

Does that answer your question?
 
porsupuesto3798
Thanks Received: 2
Vinny Gambini
Vinny Gambini
 
Posts: 16
Joined: May 03rd, 2012
 
 
 

Re: Q19 - The coach of the Eagles

by porsupuesto3798 Thu May 31, 2012 11:16 am

When I saw this problem, I thought that the conclusion was incorrect because it fails to consider the circumstance of a draw.
Not lost is not equal to win, a draw could also be a possibility. However, nothing in the five answers is close to this one.
That's how I choose (A) which appears to talk about what I mentioned but actually did not.

In fact, there is another problem with this stimulus, that is what happened in the past does not necessarily repeat in the future.
 
eunjung.shin
Thanks Received: 2
Jackie Chiles
Jackie Chiles
 
Posts: 40
Joined: December 08th, 2011
 
 
 

Re: Q19 - The coach of the Eagles

by eunjung.shin Mon Jun 11, 2012 3:09 am

Why is E wrong? I was debating between D and E and bc the conclusions says "this sort of info is valable" - thuis refering to best combo of players done by this computer anaysis, i thought E ma tched the flaw better.


What are your thoughts?

Thanks!
User avatar
 
ManhattanPrepLSAT1
Thanks Received: 1909
Atticus Finch
Atticus Finch
 
Posts: 2851
Joined: October 07th, 2009
 
 
 

Re: Q19 - The coach of the Eagles

by ManhattanPrepLSAT1 Mon Jun 25, 2012 3:24 pm

eunjung.shin Wrote:Why is E wrong? I was debating between D and E and bc the conclusions says "this sort of info is valable"

Not sure if it was a typo or intentional, but your interpretation of this one says that there are multiple "conclusions." This is what probably led to you being tempted with answer choice (E).

If you interpret the argument as concluding that this sort of information is valuable, then one might be tempted with answer choice (E) that points out that the information did not provided facts beyond what could be determined without a computer. But the argument is not making this conclusion. There is no evidence to support the claim that this information is valuable and so cannot be the thrust of the argument. Instead, the argument is trying to assert a claim about Jennifer's presence in the game.

Another issue with answer choice (E) is that is says that the information was already known. The stimulus says, however, that no computer was needed to discover this information - there's a difference between those two ideas.

Hope that helps!
 
nflamel69
Thanks Received: 16
Atticus Finch
Atticus Finch
 
Posts: 162
Joined: February 07th, 2011
 
 
 

Re: Q19 - The coach of the Eagles

by nflamel69 Tue Sep 04, 2012 6:32 pm

So technically A is still wrong because it confused between necessary and sufficient, it described the reasoning backwards. But even if it corrected, it would still be wrong because then it would be a correct reasoning? I totally get this. But it still sounds weird when I reasoned it out
 
shirando21
Thanks Received: 16
Atticus Finch
Atticus Finch
 
Posts: 280
Joined: July 18th, 2012
 
 
 

Re: PT 54, S2, Q19 - The coach of the Eagles...

by shirando21 Sun Nov 11, 2012 11:00 pm

mattsherman Wrote:The answer would be correct if it said

(A) infers from the fact that a certain factor is necessary for a result that the absence of that factor is sufficient for the opposite result

really close, huh?


so close and so subtle...
 
amil91
Thanks Received: 5
Elle Woods
Elle Woods
 
Posts: 59
Joined: August 02nd, 2013
 
 
 

Re: Q19 - The coach of the Eagles

by amil91 Mon Nov 11, 2013 6:17 pm

mattsherman Wrote:
eunjung.shin Wrote:Why is E wrong? I was debating between D and E and bc the conclusions says "this sort of info is valable"

Not sure if it was a typo or intentional, but your interpretation of this one says that there are multiple "conclusions." This is what probably led to you being tempted with answer choice (E).

If you interpret the argument as concluding that this sort of information is valuable, then one might be tempted with answer choice (E) that points out that the information did not provided facts beyond what could be determined without a computer. But the argument is not making this conclusion. There is no evidence to support the claim that this information is valuable and so cannot be the thrust of the argument. Instead, the argument is trying to assert a claim about Jennifer's presence in the game.

Another issue with answer choice (E) is that is says that the information was already known. The stimulus says, however, that no computer was needed to discover this information - there's a difference between those two ideas.

Hope that helps!

In addition to this, the stimulus does not discuss the value of the computer analyses, it simply discusses the value of the actual information.
nflamel69 Wrote:So technically A is still wrong because it confused between necessary and sufficient, it described the reasoning backwards. But even if it corrected, it would still be wrong because then it would be a correct reasoning? I totally get this. But it still sounds weird when I reasoned it out

I think on it's face it appears to switch the words sufficient and necessary causing us to think it is incorrect because it doesn't fit with the stimulus, but that's not actually the case. I'd say for most people we read this answer choice as referring to Jennifer not playing as being the factor discussed, in which case A did reverse the words sufficient and necessary, but aside from that, A does not represent a logical flaw as stated nor does the stimulus in terms of conditional logic. But if you ignore your notion of what factor it is discussing, it is OK as written. For example, If A then B - sufficient factor is A and the result is B and B is necessary. The absence of that factor (A) is necessary for the opposite of the result (B). If -B then -A - sufficient factor in this case is the opposite of the result in the previous case and -A is the absence of the factor that was sufficient in the previous case but is now necessary for this opposite result (-B). So A is actually OK logically, I think for most people it would just appear to have switched around the sufficient and necessary because of the way we read the stimulus, at least it did for me. Point being, the flaw in this argument, IMO, is more of a common sense flaw than a logical one - that things that have happened in the past will certainly continue to happen in the future, and no so much a violation of a formal logic rule.
User avatar
 
Mab6q
Thanks Received: 31
Atticus Finch
Atticus Finch
 
Posts: 290
Joined: June 30th, 2013
 
 
 

Re: Q19 - The coach of the Eagles...

by Mab6q Sun Mar 16, 2014 5:23 pm

mattsherman Wrote:
zainrizvi Wrote:mshermn wrote:
The answer would be correct if it said

(A) infers from the fact that a certain factor is necessary for a result that the absence of that factor is sufficient for the opposite result

really close, huh?



I don't think that would be right though. Right now it says:
L - > Jnp
The contrapositive being,
Jp -> W

Isn't that valid reasoning?


Hey zainrizvi! Nice work, you are absolutely correct! My explanation was incorrect. I should have caught that. Answer choice (A) would not have been correct had it been worded as I suggested since it still would have been valid reasoning. I think I was trying to describe what was happening in the argument and lost sight of the fact that we were looking for the error in the reasoning. The use of the contrapositive, which is essentially one element of the reasoning structure and that is highlighted in answer choice (A) is not a reasoning error and so cannot answer the question about the argument's vulnerability.

changsoyeon Wrote:The second question I had was, isn't there also a correlation-causation issue going on here? Just because in the past the team has lost only when Jennifer wasn't palying, you can't conclude that Jennifer's presence ENSURES (causation word) that they will win. Something else could have caused the team to lose and it just happened to be that on those days, Jennifer wasn't playing, right? I went to look for an answer choice that had a correlation-causation issue, but I wasn't able to find any, and when I read (D), the word "coincided" popped out at me, and so I knew that had to be the flaw.

So I don't see a correlation/causation issue going on here. "Ensures" is still a word that implies correlation.

I must say, I think I do see the correlation causation issue. The author says that because we had this occurrence in the past, we will continue to see it. That means that correlation in the past will continue to the future, meaning he thinks that it is not correlation but causation, that is why he is so sure. Thoughts?

Also, i dont think we can diagram the jenifer playing premise as a conditional because we dont know that it is. We only know about what has been the instance in the past. If we do diagram it as a conditional, then the author's reasoning is sound by nature or negating the necessary condition to get Eagles will win.
"Just keep swimming"
 
matthughes2
Thanks Received: 0
Vinny Gambini
Vinny Gambini
 
Posts: 8
Joined: November 18th, 2015
 
 
 

Re: Q19 - The coach of the Eagles

by matthughes2 Wed Nov 18, 2015 1:24 pm

I think you're all getting stuck on the conditional logic. Answer choice A displays a correct contrapositive of the relationship posited in the prompt, but doesn't address a fundamental weakness which is that just because Jennifer did not play in each game that the Eagles lost, her presence does not ensure a win in future games. If that were the case, they'd never let her graduate!

The logic in the prompt goes:

~EW > ~JP
the contra:
JP > EW

A) is "correct" only insofar as it shows the proper negation of this relationship ("certain factor sufficient for a result" translates to "Jennifer not playing sufficient for Eagles to not win" and "absence of that factor is necessary for the opposite result" means "Jennifer playing necessary for a win").

Where you run into trouble is that the prompt says that Jennifer being in the game ENSURES a win. While the logic might dictate that Jennifer playing is necessary for the team to win (as dictated in the negation displayed in A), that doesn't mean that just because she plays that they WILL win.

D) addresses this (in a crappy, roundabout way) by saying that the speaker presumes that because Jennifer not playing in the past has resulted in losses, that Jennifer playing means that the team WILL win ALL future games (this is the meaning of "ensures that the Eagles will win").

The devil is in the details, guys. The problem with A is in the prompt, not the answer choice. The problem with A isn't the logic; it's that the logic doesn't address the fact that the speaker confuses what's required to win with what will ensure a win in the future. If every time I wear a green shirt, I get a girl's number, that doesn't mean that wearing a green shirt means I will inevitably get a girl's number.
 
haeeunjee
Thanks Received: 15
Jackie Chiles
Jackie Chiles
 
Posts: 37
Joined: May 05th, 2016
 
 
 

Re: Q19 - The coach of the Eagles

by haeeunjee Fri May 20, 2016 2:12 pm

I am a little confused about why all the explanations above are even about conditionals... Why are we giving conditional weight to the data that the author presents? The data only shows a mild correlation: The team lost only when Jennifer was not playing. The "only when J wasn't playing" seems at first to signal a necessary condition for a conditional statement, but... The author isn't saying as a definite premise: Whenever the team loses, J doesn't play. In that case, it would be a conditional. (I guess L --> ~J and CP: J --> ~L).

But I took it to mean that the author is just presenting data that shows these two data categories (Jennifer not playing, team losing) to be correlated. No conditionals involved, since wouldn't that mean we're taking for granted L --> ~J holds water / is true / a premise that the author posits? The author never gives us a conditional as a premise. When we make a conditional from the (weak) data that the author presents, it seems that we are making an additional leap of logic for the author.

(A) says that it is a flaw that the author "infers from the fact that a certain factor is sufficient" to mean that the absence of that factor is necessary for the opposite result. First of all, like people said, this describes a contrapositive, which isn't a logical fallacy anyways. But further, this says "infers from the fact that a certain factor is sufficient." Wait hold up, so now it's a FACT that L is sufficient for ~J? Again, the author never posits that "fact" nor gives a conditional to begin with. Just some correlative data.

I chose (D) because I thought it was a combo of a temporal fallacy and a correlation fallacy. And yes, I understand that "ensure" may not be a causal indicator, and that it actually may be a sufficiency indicator (If it rains, then Sue wears a jacket = Rain ensures that Sue will wear a jacket). In that case, I guess I saw it as a temporal/correlation combo fallacy: Just cause a correlation has held up in the past, doesn't mean it'll hold up in the future. Definitely doesn't mean it has to hold up in the future. So in this case, I guess "ensures" doesn't even have to be interpreted as indicating causal or conditional for it to be wrong - assuming a correlation has any "ensuring"/guaranteeing power into the future is simply wrong.

Any thoughts on this reasoning? I was also stuck on (A) and (D) but ended up not even diagramming and just going with my gut instinct that screamed temporal fallacy.
 
AnnaT620
Thanks Received: 0
Elle Woods
Elle Woods
 
Posts: 51
Joined: May 25th, 2020
 
 
 

Re: Q19 - The coach of the Eagles

by AnnaT620 Thu Feb 25, 2021 10:33 am

Hi Team

I really struggled with this one - I initially liked both D and E, but then used the strategy discussed in the course of comparing the answer choices against the stimulus, and the premise / conclusion.

I obviously incorrectly understood that the conclusion was "this sort of information is valuable" rather than relating to Jennifer being required to win the game - is there a way that I can test this going forward?

Many thanks!!
 
Laura Damone
Thanks Received: 94
Atticus Finch
Atticus Finch
 
Posts: 468
Joined: February 17th, 2011
 
 
 

Re: Q19 - The coach of the Eagles

by Laura Damone Tue Mar 02, 2021 4:25 pm

I think flaw fluency is the key to this question. When you are fluent in the flaws that appear on the LSAT, the word "ensure" in the last line should light up your brain because it's conditional logic language. When I read this one, I see "ensure" and I go "oh REALLY? She's gonna ENSURE victory? Yeah right." Then I go back digging in the argument to figure out what premise led to the bogus conclusion that Jennifer is sufficient to ensure a win. The premise I find is the data: The team has only lost when she's not playing. That means this argument assumes that what's been true in the past will continue to be true in the future: a Past vs. Future flaw!

In terms of testing whether Jenny ensuring the win is the conclusion vs. the info being valuable, that's a tough one. The "therefore" test only works well if both claims you're testing are part of the argument core. In this case, the info being valuable is a red herring, and it doesn't support the conclusion that Jenny will ensure victory.

However, answer choice E can still be eliminated because of a subtle detail creep. Although the argument establishes that "no computer was needed to discover this information," it also establishes that "The [computer] analysis revealed that the team has only lost when Jennifer was not playing." In other words, the computer analysis discovered this fact. Just because it could have been discovered in other ways doesn't mean it was. Thus, when E says "computer analysis provided no facts beyond what was already known," it's changing the story just enough to make the answer wrong.

Hope this helps!
Laura Damone
LSAT Content & Curriculum Lead | Manhattan Prep